10
$\begingroup$

I’m trying to understand solvability of $$ L_X u = f $$ for a vector field $X$ on a manifold, and then compare this with some related linear problems for left-invariant forms on a Lie group (where things look much more algebraic).


0-form case (functions on a manifold)

Let $M$ be a smooth manifold and let $X$ be a complete vector field on $M$ with flow $\phi_t$. Given a smooth function $f \in C^\infty(M)$, I want to understand when there exists a smooth function $u$ with $$ L_X u = X(u) = f. $$

Along each orbit of the flow of $X$, this is just the ODE $$ \frac{d}{dt}\big(u(\phi_t(x))\big) = f(\phi_t(x)). $$

If the orbit through $x$ is periodic of period $T>0$, i.e. $\phi_T(x)=x$, then integrating over one period gives $$ 0 = u(\phi_T(x)) - u(x) = \int_0^T f(\phi_t(x))\,dt, $$ so I get the familiar necessary condition $$ \int_0^T f(\phi_t(x))\,dt = 0 \quad \text{for every periodic orbit of } X. $$

Equivalently:

For every periodic orbit $\gamma$ of $X$, $$ \int_\gamma f\, dt = 0.$$

So for the $0$-form equation $L_X u = f$, the “zero integral on periodic orbits” condition is clearly necessary.

What I am not sure about is whether this condition is also sufficient, or whether there can be more subtle global obstructions.

So my main question is:

Question A.
Let $X$ be a complete vector field on a smooth manifold $M$, and let $f \in C^\infty(M)$. Suppose that for every periodic orbit $\gamma$ of $X$ one has $$\int_\gamma f\,dt = 0.$$ Is this condition sufficient to guarantee the existence of a global smooth solution $u$ to $$L_X u = f \, ?$$ If not, what additional obstructions can occur, or under what extra hypotheses on $X$ or $M$ does this condition become sufficient?

If every orbit is a circle, the answer is Yes. In the circle case, if the equation has no solution, the periodic orbit integrals will explicitly tell you so (by being non-zero). The condition $\int_\gamma f = 0$ is now perfectly perfectly aligned with solvability.


Similar Lie algebra problems on a Lie group

Now let $G$ be a connected Lie group with Lie algebra $\mathfrak{g}$. I identify

  • $\mathfrak{g}$ with the left-invariant vector fields on $G$,
  • $\mathfrak{g}^*$ with the left-invariant $1$-forms on $G$,
  • $\Lambda^2 \mathfrak{g}^*$ with the left-invariant $2$-forms on $G$.

Fix $X \in \mathfrak{g}$. In these left-invariant settings, the equations for $1$-forms and $2$-forms reduce to purely linear problems on finite-dimensional vector spaces, where I think I can describe the solvability conditions quite explicitly.

(1) Left-invariant $1$-forms

If $\beta \in \mathfrak{g}^*$ is left-invariant, then $$ L_X \beta = \operatorname{ad}_X^*(\beta), $$ so solving $$ L_X \beta = \alpha, \qquad \alpha \in \mathfrak{g}^* $$ is equivalent to solving the linear algebra equation $$ \operatorname{ad}_X^*(\beta) = \alpha \quad \text{on } \mathfrak{g}^*. $$

Necessary condition

If $Y \in \mathfrak{g}$ is left-invariant, then $\beta(Y)$ is a constant function on $G$. Using the identity $$ L_X \beta(Y) = X\cdot(\beta(Y)) - \beta([X,Y]) $$ and the fact that $X\cdot(\beta(Y)) = 0$ for left-invariant $X,\beta,Y$, we get $$ (L_X \beta)(Y) = -\beta([X,Y]). $$

If $L_X \beta = \alpha$, then evaluating at $Y$ gives $$ \alpha(Y) = (L_X \beta)(Y) = -\beta([X,Y]). $$

In particular, if $Y$ commutes with $X$, i.e. $[X,Y]=0$, then $$ \alpha(Y) = 0. $$

So a necessary condition is: $$ \alpha(Y) = 0 \quad \text{for all } Y \in \mathfrak{g} \text{ with } [X,Y]=0, $$ i.e. $\alpha$ vanishes on the centralizer $$ \ker(\operatorname{ad}_X) = \{Y \in \mathfrak{g} : [X,Y]=0\}. $$

Sufficiency via linear algebra

Let $A = \operatorname{ad}_X : \mathfrak{g} \to \mathfrak{g}$. Then $A^* = \operatorname{ad}_X^* : \mathfrak{g}^* \to \mathfrak{g}^*$.

In finite-dimensional linear algebra there is the standard identity $$ \operatorname{im}(A^*) = (\ker A)^\circ, $$ where $(\ker A)^\circ \subset \mathfrak{g}^*$ is the annihilator of $\ker A$, i.e. $$ (\ker A)^\circ = \{\lambda \in \mathfrak{g}^* \mid \lambda(v) = 0 \ \text{for all } v \in \ker A\}. $$

Sketch of proof:

  • If $\lambda \in \operatorname{im}(A^*)$, write $\lambda = A^*(\beta)$. Then for any $v \in \ker A$, $$ \lambda(v) = \beta(A v) = \beta(0) = 0, $$ so $\lambda \in (\ker A)^\circ$. Hence $\operatorname{im}(A^*) \subset (\ker A)^\circ$.

  • Since $\mathfrak{g}$ is finite-dimensional, $$ \dim(\operatorname{im}(A^*)) = \dim(\mathfrak{g}^*) - \dim(\ker A^*) = \dim\mathfrak{g} - \dim(\ker A), $$ while $$ \dim((\ker A)^\circ) = \dim\mathfrak{g}^* - \dim(\ker A) = \dim\mathfrak{g} - \dim(\ker A). $$ Thus the two subspaces $\operatorname{im}(A^*)$ and $(\ker A)^\circ$ have the same dimension, and with the inclusion above this gives equality: $$ \operatorname{im}(A^*) = (\ker A)^\circ. $$

Applied to $A = \operatorname{ad}_X$, this says: $$ \operatorname{im}(\operatorname{ad}_X^*) = \{\alpha \in \mathfrak{g}^* : \alpha(Y) = 0 \text{ for all } Y \in \ker(\operatorname{ad}_X)\}. $$

Therefore the equation $$ \operatorname{ad}_X^*(\beta) = \alpha $$ is solvable in $\beta \in \mathfrak{g}^*$ if and only if $$ \alpha(Y) = 0 \quad \text{for all } Y \in \mathfrak{g} \text{ with } [X,Y]=0. $$

So in this left-invariant $1$-form setting, the condition “$\alpha$ vanishes on all commuting directions” is both necessary and sufficient.


(2) Left-invariant $2$-forms

Now let me look at the analogous problem for left-invariant $2$-forms.

Left-invariant $2$-forms on $G$ correspond to elements of $\Lambda^2 \mathfrak{g}^*$. If $\beta \in \Lambda^2 \mathfrak{g}^*$ is left-invariant, then $L_X$ acts as the dual of the induced map $$ \operatorname{ad}_X : \Lambda^2 \mathfrak{g} \to \Lambda^2 \mathfrak{g}, $$ defined on decomposable bivectors by $$ \operatorname{ad}_X (Y \wedge Z) = [X,Y] \wedge Z + Y \wedge [X,Z], $$ and extended linearly.

Thus the equation $$ L_X \beta = \alpha, \qquad \alpha \in \Lambda^2 \mathfrak{g}^* $$ is equivalent to the linear algebra equation $$ \operatorname{ad}_X^*(\beta) = \alpha \quad \text{on } \Lambda^2 \mathfrak{g}^*, $$ where now $\operatorname{ad}_X^*$ is the dual operator to $$ \operatorname{ad}_X : \Lambda^2 \mathfrak{g} \to \Lambda^2 \mathfrak{g}. $$

Necessary condition

Let $B = \operatorname{ad}_X : \Lambda^2 \mathfrak{g} \to \Lambda^2 \mathfrak{g}$. If $W \in \Lambda^2 \mathfrak{g}$ lies in $\ker B$, then for any $\beta$ we have $$ (B^*\beta)(W) = \beta(BW) = \beta(0) = 0. $$

In particular, for a decomposable element $W = Y \wedge Z$, the condition $W \in \ker B$ reads $$ \operatorname{ad}_X (Y \wedge Z) = [X,Y] \wedge Z + Y \wedge [X,Z] = 0. $$

So if $L_X \beta = \alpha$ (equivalently $B^* \beta = \alpha$), then for all pairs $(Y,Z)$ with $$ 0 = [X,Y] \wedge Z + Y \wedge [X,Z], $$ we must have $$ \alpha(Y,Z) = \alpha(Y \wedge Z) = 0. $$

That is, a necessary condition is:

For all pairs $(Y,Z) \in \mathfrak{g} \times \mathfrak{g}$ such that $$ 0 = [X,Y] \wedge Z + Y \wedge [X,Z],$$ we have $$\alpha(Y,Z) = 0. $$

Sufficiency via the same linear algebra identity

Again, $B$ is a linear map on a finite-dimensional vector space $\Lambda^2 \mathfrak{g}$, so the same linear algebra argument as before gives $$ \operatorname{im}(B^*) = (\ker B)^\circ, $$ where $(\ker B)^\circ \subset (\Lambda^2 \mathfrak{g})^* \cong \Lambda^2 \mathfrak{g}^*$ is the annihilator of $\ker B$.

Thus the equation $$ \operatorname{ad}_X^*(\beta) = \alpha $$ on $\Lambda^2 \mathfrak{g}^*$ is solvable if and only if $$ \alpha(W) = 0 \quad \text{for all } W \in \ker\big(\operatorname{ad}_X : \Lambda^2 \mathfrak{g} \to \Lambda^2 \mathfrak{g}\big), $$ i.e. iff $$ \alpha(Y,Z) = 0 \ \text{whenever } [X,Y] \wedge Z + Y \wedge [X,Z] = 0. $$

So in this left-invariant $2$-form setting, the condition “$\alpha$ vanishes on all pairs $(Y,Z)$ with $[X,Y] \wedge Z + Y \wedge [X,Z] = 0$” is again both necessary and sufficient.


Back to the $0$-form case

In the left-invariant $1$- and $2$-form problems on a Lie group, the solvability conditions for $L_X \beta = \alpha$ reduce cleanly to linear algebra: $\alpha$ must lie in the image of the appropriate dual operator, which can be expressed concretely as annihilating the kernel of $\operatorname{ad}_X$ (on $\mathfrak{g}$ or on $\Lambda^2 \mathfrak{g}$, respectively).

In the original $0$-form problem on a general manifold, the natural necessary condition is the “zero integral over each periodic orbit” condition. My question is whether, in that functional setting, $$ \int_\gamma f\,dt = 0 \ \ \text{for every periodic orbit } \gamma $$ already gives a full characterization of solvability of $L_X u = f$, or whether additional global obstructions can arise beyond the periodic orbits.

$\endgroup$

1 Answer 1

7
$\begingroup$

Consider the following example that shows the answer to Question A in its basic form is no.

Let $X = (1,\sqrt{2})$ be a constant vector field on $\mathbb{R}^2$, which descends to a vector field on the torus $T^2 = \mathbb{R}^2 / \mathbb{Z}^2$. Let $f \equiv 1$ on $T^2$. Since $\sqrt{2} \notin \mathbb{Q}$, the flow $\phi_t$ has no closed orbits, so the assumption holds for trivial reasons. But $\mathcal{L}_X u = f$ is easily seen to have no solutions - just because $\mathcal{L}_X u(p) = 0$ at the point $p \in T^2$ where $u$ achieves its maximum.

You can probably fix this problem by strengthening the assumption in some way, so that you avoid the chaotic behavior of the flow. I haven't read the whole question, so I cannot say more, but I hope this is a good start.

$\endgroup$

You must log in to answer this question.

Start asking to get answers

Find the answer to your question by asking.

Ask question

Explore related questions

See similar questions with these tags.